Difference between revisions of "2021 Fall AMC 12A Problems/Problem 8"

(Solution)
Line 4: Line 4:
 
<math>\textbf{(A)}\ 1 \qquad\textbf{(B)}\ 2 \qquad\textbf{(C)}\ 37 \qquad\textbf{(D)}\ 74 \qquad\textbf{(E)}\ 2886</math>
 
<math>\textbf{(A)}\ 1 \qquad\textbf{(B)}\ 2 \qquad\textbf{(C)}\ 37 \qquad\textbf{(D)}\ 74 \qquad\textbf{(E)}\ 2886</math>
  
==Solution==
+
==Solution 1==
  
 
By the definition of least common mutiple, we take the greatest powers of the prime numbers of the prime factorization of all the numbers, that we are taking the <math>\text{lcm}</math> of. In this case, <cmath>M = 2^4 \cdot 3^3 \cdot 5^2 \cdot 7 \cdot 11 \cdot 13 \cdot 17 \cdot 19 \cdot 23 \cdot 29.</cmath> Now, using the same logic, we find that <cmath>N = M \cdot 2 \cdot 37,</cmath> because we have an extra power of <math>2</math> and an extra power of <math>37.</math> Thus, <math>\frac{N}{M} = 2\cdot 37 = 74</math>. Thus, our answer is <math>\boxed {\textbf{(D)}}.</math>
 
By the definition of least common mutiple, we take the greatest powers of the prime numbers of the prime factorization of all the numbers, that we are taking the <math>\text{lcm}</math> of. In this case, <cmath>M = 2^4 \cdot 3^3 \cdot 5^2 \cdot 7 \cdot 11 \cdot 13 \cdot 17 \cdot 19 \cdot 23 \cdot 29.</cmath> Now, using the same logic, we find that <cmath>N = M \cdot 2 \cdot 37,</cmath> because we have an extra power of <math>2</math> and an extra power of <math>37.</math> Thus, <math>\frac{N}{M} = 2\cdot 37 = 74</math>. Thus, our answer is <math>\boxed {\textbf{(D)}}.</math>
  
 
~NH14
 
~NH14
 +
 +
== Solution 2 ==
 +
First, we compute <math>M</math>. We have
 +
<cmath>
 +
\[
 +
M = 2^4 \cdot 3^3 \cdot 5^2 \cdot 7 \cdot 11 \cdot 13 \cdot 17 \cdot 19 \cdot 23 \cdot 29.
 +
\]
 +
</cmath>
 +
 +
Second, we compute <math>N</math>. We have
 +
<cmath>
 +
\[
 +
N = M \cdot 2 \cdot 37 .
 +
\]
 +
</cmath>
 +
 +
Therefore, <math>\frac{N}{M} = 2 \cdot 37 = 74</math>.
 +
 +
Therefore, the answer is <math>\boxed{\textbf{(D) }74}</math>.
 +
 +
~Steven Chen (www.professorchenedu.com)
 +
  
 
==See Also==
 
==See Also==
  
 
{{AMC12 box|year=2021 Fall|ab=A|num-b=7|num-a=9}}
 
{{AMC12 box|year=2021 Fall|ab=A|num-b=7|num-a=9}}

Revision as of 21:38, 25 November 2021

Problem

Let $M$ be the least common multiple of all the integers $10$ through $30,$ inclusive. Let $N$ be the least common multiple of $M,32,33,34,35,36,37,38,39,$ and $40.$ What is the value of $\frac{N}{M}?$

$\textbf{(A)}\ 1 \qquad\textbf{(B)}\ 2 \qquad\textbf{(C)}\ 37 \qquad\textbf{(D)}\ 74 \qquad\textbf{(E)}\ 2886$

Solution 1

By the definition of least common mutiple, we take the greatest powers of the prime numbers of the prime factorization of all the numbers, that we are taking the $\text{lcm}$ of. In this case, \[M = 2^4 \cdot 3^3 \cdot 5^2 \cdot 7 \cdot 11 \cdot 13 \cdot 17 \cdot 19 \cdot 23 \cdot 29.\] Now, using the same logic, we find that \[N = M \cdot 2 \cdot 37,\] because we have an extra power of $2$ and an extra power of $37.$ Thus, $\frac{N}{M} = 2\cdot 37 = 74$. Thus, our answer is $\boxed {\textbf{(D)}}.$

~NH14

Solution 2

First, we compute $M$. We have \[ M = 2^4 \cdot 3^3 \cdot 5^2 \cdot 7 \cdot 11 \cdot 13 \cdot 17 \cdot 19 \cdot 23 \cdot 29. \]

Second, we compute $N$. We have \[ N = M \cdot 2 \cdot 37 . \]

Therefore, $\frac{N}{M} = 2 \cdot 37 = 74$.

Therefore, the answer is $\boxed{\textbf{(D) }74}$.

~Steven Chen (www.professorchenedu.com)


See Also

2021 Fall AMC 12A (ProblemsAnswer KeyResources)
Preceded by
Problem 7
Followed by
Problem 9
1 2 3 4 5 6 7 8 9 10 11 12 13 14 15 16 17 18 19 20 21 22 23 24 25
All AMC 12 Problems and Solutions